ChaseDream
搜索
12下一页
返回列表 发新帖
00:00:00

Which of the following, if true, provides evidence that most logically completes the argument below?

According to a widely held economic hypothesis, imposing strict environmental regulations reduces economic growth. This hypothesis is undermined by the fact that the states with the strictest environmental regulations also have the highest economic growth. This fact does not show that environmental regulations promote growth, however, since ______.

正确答案: A

更多相关帖子

524

帖子

15

好友

4712

积分

ChaseDream

注册时间
2003-03-17
精华
8
解析
查看: 8824|回复: 17
打印 上一主题 下一主题

gwd29-8答案似乎有争议

[复制链接]
楼主
发表于 2006-12-6 06:50:00 | 只看该作者

gwd29-8答案似乎有争议

GWD-29-Q8

Which of the following, if true, provides evidence that most logically completes the argument below?

 

According to a widely held economic hypothesis, imposing strict environmental regulations reduces economic growth.  This hypothesis is undermined by the fact that the states with the strictest environmental regulations also have the highest economic growth.  This fact does not show that environmental regulations promote growth, however, since ______.

 

  1. those states with the strictest environmental regulations invest the most in education and job training
  2. even those states that have only moderately strict environmental regulations have higher growth than those with the least-strict regulations
  3. many states that are experiencing reduced economic growth are considering weakening their environmental regulations
  4. after introducing stricter environmental regulations, many states experienced increased economic growth
  5. even those states with very weak environmental regulations have experienced at least some growth

答案给的是A,但我觉得投资什么并没有说明regulation和growth的关系,相反我觉得e很好的说明了。

假设是:严格的法规--〉增长下降

事实是:严格法规有高增长

然而,事实并没有显示严格法规促进了增长,所以我们给一个严格法规没有促进增长就很好的说明了问题,所以我觉得此话的双重否定不严格的法规也有增长也就是e.说明了问题

请nn指点一下。。。欢迎大家讨论一下拉

沙发
发表于 2006-12-6 21:45:00 | 只看该作者
以下是引用yl1004在2006-12-6 6:50:00的发言:

GWD-29-Q8

Which of the following, if true, provides evidence that most logically completes the argument below?

 

 

According to a widely held economic hypothesis, imposing strict environmental regulations reduces economic growth.  This hypothesis is undermined by the fact that the states with the strictest environmental regulations also have the highest economic growth.  This fact does not show that environmental regulations promote growth, however, since ______.

 

 

  1. those states with the strictest environmental regulations invest the most in education and job training
  2. even those states that have only moderately strict environmental regulations have higher growth than those with the least-strict regulations
  3. many states that are experiencing reduced economic growth are considering weakening their environmental regulations
  4. after introducing stricter environmental regulations, many states experienced increased economic growth
  5. even those states with very weak environmental regulations have experienced at least some growth

答案给的是A,但我觉得投资什么并没有说明regulation和growth的关系,相反我觉得e很好的说明了。

假设是:严格的法规--〉增长下降

事实是:严格法规有高增长

然而,事实并没有显示严格法规促进了增长,所以我们给一个严格法规没有促进增长就很好的说明了问题,所以我觉得此话的双重否定不严格的法规也有增长也就是e.说明了问题

请nn指点一下。。。欢迎大家讨论一下拉

我是这么认为的,读题目前面:imposing strict environmental regulations ==>reduces economic growth

而后面给的fact是:states with the strictest environmental regulations ==> have the highest economic growth

我觉得两者是有区别的,一个是说施行降低增长,而另一个是说有最严格的法规的州,感觉两个说的不是一回事.这样,就可以看出A选项说那些州的增长是因为 invest the most in education and job training,而不是因为impose regulation了.

而E答案,我觉得明显是不对的,结论就是要推翻说impose regulation can stimulate economy, for me, e is out of scope

板凳
 楼主| 发表于 2006-12-7 06:54:00 | 只看该作者
还是不明白,总觉得第一个选项好像不能解释什么问题。
地板
发表于 2006-12-7 07:11:00 | 只看该作者

可是我觉得e不能反映什么,因为最后题目说的是:这个事实不能反映严格法规能够推翻前面的假设...所以就要解释前面的这个事实是为什么原因...而不是说那些weak regulations的国家

5#
发表于 2006-12-7 07:24:00 | 只看该作者

继续补充我的观点:

它是说impose 会reduce,而那些有些最严格法规的人经济增长是最快的,那也许它们增长的可能比原来还要快,比如原来是9%,然后有了那些严格的法规后只有8%了,而相对于其他的国家还是highest的.为什么这些国家具有如此高的增长率呢?因为它们把钱投资到国民教育去啦!!!所以那么高~

不知道gg现在读懂了没...

6#
 楼主| 发表于 2006-12-9 03:09:00 | 只看该作者

似乎有道理,谢谢楼上的mm

7#
发表于 2006-12-9 10:29:00 | 只看该作者

Choice A shows that it is investment in education and job training that leads to the highest growth, hence weaken the possible causal relationship between environmental regulation and the economic growth.

If you treat the question as a weakening question, you know how to handle it.

E is at least no the best choice. If environmental regulation contributes growth, it is possible weak regulation to have weak growth, but some growth. So basically the E does not provide the best completion of the reasoning in the passage.


[此贴子已经被作者于2006-12-9 10:34:18编辑过]
8#
发表于 2007-1-13 11:42:00 | 只看该作者
up
9#
发表于 2007-5-13 18:04:00 | 只看该作者
ding
10#
发表于 2007-6-19 13:57:00 | 只看该作者

The question talks about fact has "strictest/most", A is best since it also has "strictest/the most". IMO

您需要登录后才可以回帖 登录 | 立即注册

Mark一下! 看一下! 顶楼主! 感谢分享! 快速回复:

手机版|ChaseDream|GMT+8, 2024-9-20 06:52
京公网安备11010202008513号 京ICP证101109号 京ICP备12012021号

ChaseDream 论坛

© 2003-2023 ChaseDream.com. All Rights Reserved.

返回顶部